Những câu hỏi liên quan
❤  Hoa ❤
Xem chi tiết

2x + 2y + z = 4(1)
A = 2xy + yz + xz(2)
(1) z=2c<=>x+y=2-c($)
(2)<=>2xy+2yc+2cx=A
A=2B<=>xy +(x+y).c=B
xy=B-c(2-c)
($:%)=> ton tai nghiem x,y
(c-2)^2≥4[B+c(c-2)]
c^2-4c+4≥4B+4c^2-8c
-3c^2+4c≥4B-4
-3(c^2-2.2/3c+4/9)≥4B-4-4/3
-3(c-2/3)^2≥4B-16/3
=> B≤4/3
A≤8/3
dang thuc khi c=2/3; z=1/3
x=y=2/3

Trần Nhật Dương
9 tháng 5 2019 lúc 20:45

A=2xy+yz+xzA=2xy+yz+xz

=2xy+y(4−2x−2y)+x(4−2x−2y)=2xy+y(4−2x−2y)+x(4−2x−2y)

=−2x2−2xy+4x−2y2+4y=−2x2−2xy+4x−2y2+4y

=[−(x2+2xy+y2)+83(x+y)−169]−(x2−43x+49)−(y−43y+49)+83=[−(x2+2xy+y2)+83(x+y)−169]−(x2−43x+49)−(y−43y+49)+83=−(x+y−43)2−(x−23)2−(y−23)2+83≤83=−(x+y−43)2−(x−23)2−(y−23)2+83≤83

Vậy Amax=83Amax=83 tại 

Nguyễn Viết Ngọc
9 tháng 5 2019 lúc 20:49

https://h.vn/hoi-dap/question/604792.html

Bn tham khảo tại đây nhé !

___G-Dragon___

Quang Nhật
Xem chi tiết
Nyatmax
26 tháng 12 2019 lúc 10:22

We have:

\(P=\Sigma_{cyc}\sqrt{2x+yz}\le\sqrt{3\left[2\left(x+y+z\right)+\Sigma_{cyc}xy\right]}\le\sqrt{3\left[4+\frac{\left(x+y+z\right)^2}{3}\right]}=4\)

Sign '=' happen when \(x=y=z=\frac{2}{3}\)

Khách vãng lai đã xóa
Thuhuyen Le
Xem chi tiết
nguyễn kiều phượng
Xem chi tiết
Hoàng Lê Bảo Ngọc
20 tháng 5 2016 lúc 10:49

Áp dụng bất đẳng thức Bunhiacopxki: 

\(P^2\le\left(1^2+1^2+1^2\right)\left(2x+2y+2z+xy+yz+xz\right)=3\left(4+xy+yz+xz\right)\)

Mặt khác ta có : \(xy+yz+xz\le x^2+y^2+z^2\le\frac{\left(x+y+z\right)^2}{3}=\frac{4}{3}\) (Dấu "=" xảy ra khi x=y=z=2/3)

=> \(P\le\sqrt{3\left(4+\frac{4}{3}\right)}=4\)khi x=y=z=2/3

Vậy Max P = 4 <=> x=y=z=2/3

Chu Nguyễn Gia Huy
19 tháng 5 2020 lúc 12:25

hjhhogf hgghi huiio 

Khách vãng lai đã xóa
Thu Cúc
Xem chi tiết
Nguyễn Huy Hoàng
Xem chi tiết
Lê Thị Thục Hiền
19 tháng 5 2021 lúc 11:26

\(gt\Leftrightarrow\dfrac{1}{\sqrt{x}}+\dfrac{1}{\sqrt{y}}+\dfrac{1}{\sqrt{z}}=1\)

\(P=\dfrac{1}{xyz}\left(x\sqrt{2y^2+yz+2z^2}+y\sqrt{2x^2+xz+2z^2}+z\sqrt{2y^2+xy+2x^2}\right)\)

\(=\dfrac{1}{xyz}\left(x\sqrt{\dfrac{5}{4}\left(y+z\right)^2+\dfrac{3}{4}\left(y-z\right)^2}+y\sqrt{\dfrac{5}{4}\left(x+z\right)^2+\dfrac{3}{4}\left(x-z\right)^2}+z\sqrt{\dfrac{5}{4}\left(x+y\right)^2+\dfrac{3}{4}\left(x-y\right)^2}\right)\)

\(\ge\dfrac{1}{xyz}\left[x.\dfrac{\sqrt{5}\left(z+y\right)}{2}+y.\dfrac{\sqrt{5}\left(x+z\right)}{2}+z.\dfrac{\sqrt{5}\left(x+y\right)}{2}\right]\)

\(=\dfrac{\sqrt{5}\left(z+y\right)}{2yz}+\dfrac{\sqrt{5}\left(x+z\right)}{2xz}+\dfrac{\sqrt{5}\left(x+y\right)}{2xy}\)

\(=\dfrac{\sqrt{5}}{3}\left(1+1+1\right)\left(\dfrac{1}{x}+\dfrac{1}{y}+\dfrac{1}{z}\right)\ge\dfrac{\sqrt{5}}{3}\left(\dfrac{1}{\sqrt{x}}+\dfrac{1}{\sqrt{y}}+\dfrac{1}{\sqrt{z}}\right)^2=\dfrac{\sqrt{5}}{3}\) (bunhia)

Dấu = xảy ra khi \(x=y=z=9\)

Khôi Bùi
19 tháng 5 2021 lúc 11:30

 Thấy : \(\sqrt{2y^2+yz+2z^2}=\sqrt{\dfrac{5}{4}\left(y+z\right)^2+\dfrac{3}{4}\left(y-z\right)^2}\ge\dfrac{\sqrt{5}}{2}\left(y+z\right)>0\) 

CMTT : \(\sqrt{2x^2+xz+2z^2}\ge\dfrac{\sqrt{5}}{2}\left(x+z\right)\)  ; \(\sqrt{2y^2+xy+2x^2}\ge\dfrac{\sqrt{5}}{2}\left(x+y\right)\) 

Suy ra : \(P\ge\dfrac{1}{xyz}.\dfrac{\sqrt{5}}{2}\left[x\left(y+z\right)+y\left(x+z\right)+z\left(x+y\right)\right]\)

\(\Rightarrow P\ge\sqrt{5}\left(\dfrac{1}{x}+\dfrac{1}{y}+\dfrac{1}{z}\right)\) 

Ta có : \(\sqrt{xy}+\sqrt{yz}+\sqrt{xz}=\sqrt{xyz}\Leftrightarrow\dfrac{1}{\sqrt{x}}+\dfrac{1}{\sqrt{y}}+\dfrac{1}{\sqrt{z}}=1\) 

Mặt khác :   \(\dfrac{1}{x}+\dfrac{1}{y}+\dfrac{1}{z}\ge\dfrac{\left(\dfrac{1}{\sqrt{x}}+\dfrac{1}{\sqrt{y}}+\dfrac{1}{\sqrt{z}}\right)^2}{3}=\dfrac{1}{3}\)

Suy ra : \(P\ge\dfrac{\sqrt{5}}{3}\)

" = " \(\Leftrightarrow x=y=z=9\)

 Huyền Trang
Xem chi tiết
tthnew
15 tháng 3 2021 lúc 18:50

Ta có: 

\(2\left(2x^2+xy+2y^2\right)=3\left(x^2+y^2\right)+\left(x+y\right)^2\ge\dfrac{3}{2}\left(x+y\right)^2+1\left(x+y\right)^2=\dfrac{5}{2}\left(x+y\right)^2\)

\(\Rightarrow\sqrt{2x^2+xy+2y^2}\ge\dfrac{\sqrt{5}}{2}\left(x+y\right)\)

Gợi ý. Dùng cái trên.

 Huyền Trang
15 tháng 3 2021 lúc 17:20

Mọi người giúp mình với a :))

Hoa Hồng
Xem chi tiết
Hoa Hồng
3 tháng 3 2018 lúc 20:45

mấy bạn chuyên toán giải giùm mk bài b) giùm ạ, mk đaq rất cần

Trần Công Tiến
Xem chi tiết
Nguyễn Việt Lâm
27 tháng 4 2019 lúc 18:28

\(z=4-2x-2y\)

\(\Rightarrow A=2xy+y\left(4-2x-2y\right)+x\left(4-2x-2y\right)\)

\(A=-2y^2+4y-2x^2+4x-2xy\)

\(A=-2\left(x^2+\frac{y^2}{4}+1+xy-2x-y\right)-\frac{3}{2}\left(y^2-\frac{4}{3}y+\frac{4}{9}\right)+\frac{8}{3}\)

\(A=-2\left(x+\frac{y}{2}-1\right)^2-\frac{3}{2}\left(y-\frac{2}{3}\right)^2+\frac{8}{3}\le\frac{8}{3}\)

\(\Rightarrow A_{max}=\frac{8}{3}\) khi \(\left\{{}\begin{matrix}x=\frac{2}{3}\\y=\frac{2}{3}\\z=\frac{4}{3}\end{matrix}\right.\)